Sie sind auf Seite 1von 9

CARDIOVASCULAR PRACTICE QUESTIONS

1. Which of the following current(s) is responsible for the rapid depolarization phase of
the SA node action potential?

A. if and iNa
*B. iCa
C. iNa
D. iK and if
E. iNa and iK

2. Bert is in the middle of a marathon and his heart rate is higher than when he is resting.
This raised heart rate is due to:

*A) increase in if and iCa currents. Exercise via ↑symp increases rate of spont
depolarization of SA node which is dependent on these two currents
B) increase in iK currents
C) depolarization of the ventricular myocyte
D) increased contractility
E) activation of alpha 1 receptors.

3. Use the figure below to answer this question

A. The aortic valve opens at 2 and closes at 7


*B. The aortic valve opens at 3 and closes at 5
C. The mitral valve opens at 2 and closes at 5
D. The mitral valve closes at 3 and opens at 5
E. The mitral valve closes at 3 and opens at 7.

1
4. What is the approximate value of mean arterial pressure for the person in the above
diagram?

A. 70 mmHg
*B. 85 mmHg
C. 93 mmHg
D. 105 mmHg
E. 115 mmHg
A rough approximation of MAP = DP + (SBP – DBP)/3 DP = ~70, SBP = ~115 so
answer is ~85 mmHg

5. The left atrial pressure ‘a’ wave is due to:

*A. atrial contraction


B. regurgitation of blood from the LV to the LA during systole
C. bulging of mitral valve back into the LA during systole.
D. venous return flowing into the LA during diastole.
E. stenosis of the mitral valve.

6. Use the figure below to answer this question.

2
If a murmur was heard during C and D which increased then decreased in volume, which
of the following valve defects is most likely to be present?
A) Aortic incompetence
*B) Aortic stenosis
C) Mitral stenosis
D) Mitral incompetence
This is a systolic murmur and is heard when blood is being pushed through the
narrowed aortic valve. It is characterized by a crescendo-decrescendo sound. The
other valve abnormality causing a systolic murmur is mitral incompetence and this
is heard throughout systole probably from point 2.

7. Use the figure below to answer this question.

E’

E
F
D D’
LV
Pressure

C’
A C
B

LV volume

ABCDEF (solid line) represents the LV pressure-volume loop during a cardiac cycle.
Which of the following changes would most likely alter the loop to ABC’D’E’F ?

A. increased arteriolar constriction


B. increased contractility
*C.reduced venous compliance
D. increased venodilation
E. increased heart rate
The changes seen are typical of an increased preload. Of the factors listed only
reduced venous compliance can increase preload. Reduced venous compliance
(which happens with venoconstriction) may increase VR to the heart and increase
EDV.

8. Nitroglycerin, a drug used to treat angina is effective in this condition because it:

A. increases preload
B. increases afterload

3
C. increases contractility
*D. decreases preload
E. decreases afterload
F. decreases contractility

Nitroglycerin acts predominantly as a venodilator so it reduces preload, this reduces


the EDV and the amount of work the heart has to perform to eject blood. To a lesser
extent, E could also be correct since nitroglycerin also causes some arteriolar
vasodilation which would reduce afterload.

9. Use the figure below to answer this question.

F E
D
LV
Pressure X

A C
B

LV volume

The above represents a LV pressure – volume loop. How would increased sympathetic
activity alter this loop? Sympathetic activity would
A. Shift E downwards
B. Shift C to the right
C. Shift F to the right
*D. Increase X
E. Shift A downwards

Increase sympathetic increases contractility which shifts FA to the left. This would
then increase X which represents stroke volume.

10. Which point on the loop represents closure of the aortic valve?

A. A
B.B
C.C
D.D
*E.F

Closure of the aortic valve occurs at F

4
11. At which point on this loop is the ventricular volume greatest?

A. A
B. B
*C. D
D. E
E. F

12. Laplace Law states that

*A. if the pressure inside a sphere is kept constant, the wall tension increases as the
radius of the sphere increases.
B. the force of contraction of the myocardium is increased when the myocardial fibre
length is increased.
C. the resistance to blood flow is inversely proportional to the fourth power of the radius.
D. the resistance to blood flow is reduced when the vessels are in parallel.
E. the resistance to blood flow is increased when vessels are in series.

The other statements are correct but are not Laplace’s Law.

13. Which one of the following is required for cardiac but is not necessary for skeletal
muscle contraction?

*A. extracellular calcium


B. calcium release from the sarcoplasmic reticulum
C. troponin C
D. actin
E. myosin
Adequate Ca for skeletal muscle contraction comes from the SR.

14. Use the figure below to answer this question

The line between the two arrows is flat (isoelectric) because

5
A. atrial myocytes are depolarized whereas the ventricular myocytes have repolarized.
B. atrial myocytes and ventricular myocytes are all depolarized.
*C. the membrane potential of all the atrial and ventricular myocytes is – 85 mV.
D. the ventricular myocytes are depolarized but the atrial myocytes are resting.
E. there is no potential difference between the inside and outside of the myocytes during
this interval.

The interval between the arrows is the T-P interval when all the myocytes are in the
resting state with a RMP of ~ -85mV (some books say -90). In terms of the cardiac
cycle – the heart is in diastole.

15. Use the ECG below to calculate the mean electrical axis of the heart.

The axis is closest to

A. +30
B. -30
C. -90
*D. +180
E. +90

Measure the overall size of the QRS in the aVR : (+5 – 2 = +3) and the aVL (+2 – 6 =
negative 4). Remember the polarity of the leads! On the aVR axis measure 3 units

6
(leftwards from the centre i.e. towards the positive pole of the lead ) and on the aVL axis
measure 4 ie downwards from the centre (away from the centre – the positive lead is on
the left arm). Draw the perpendicular lines to these points and you will see that they
intersect at +180 degrees. This is sometimes recorded as -180 degrees also. An easy
mistake to make here is to count the aVL lead (4 units) in the wrong direction i.e. going
from the centre to the positive pole. If you do this your intersection will be at -90 degrees.

16. Use the following ECG – recorded from a patient, to answer this question.

This patient is most likely suffering from

A. first degree block


B. second degree block Mobitz type I
C. second degree block Mobitz type II
*D. third degree block
E. atrial fibrillation

At first glance you might think this is a second degree block Mobitz type 2 where
some of the QRS complexes fail to appear after the P wave. But notice that the PR
intervals are varying. It is a third degree block where the P waves are completely
independent of the QRS complexes. The P waves are at regular intervals, as are the
QRS complexes. More of the ECG tracing might have made it easier.

17. The oxygen consumption of a test subject was found to be 700ml/min. Pulmonary
artery oxygen content was 140 ml/L of blood and the brachial artery oxygen content was
210 ml/L of blood. His cardiac output was most likely which of the following?

A. 4.2 L/min
B. 7.0 L/min
*C. 10.0 L/min
D. 12.6 L/min
E. 30 L/min

This is Fick’s Law: CO = O2 consumption/arterio-venous O2 difference. The


arteriovenous O2 difference is 210 – 140 = 70 ml/L (the pulmonary artery has the

7
venous deoxygenated blood) and the brachial artery O2 content is equivalent to the
pulmonary venous blood in terms of oxygen content. Plug these values in and CO =
700/70 L/min

18. A 65 year old woman presents to the physician for a routine medical check up. The
physician notes that her blood pressure is 160/95 mm Hg. A drug with which of the
following actions would help reduce this woman’s blood pressure?
A. alpha 1 agonist
*B. angiotensin antagonist
C. beta 1 agonist
D. muscarinic antagonist
E. a drug that increased contractility only

An alpha agonist would cause vasoconstriction and increased SVR. Aa beta1 agonist
would increase HR and SV and increase BP, a muscarinic antagonist would increase
HR and a drug that increased contractility would increase SV. An angiotensin
antagonist would block the powerful vasoconstrictor actions of angiotensin II.

19. A 74 year old man was diagnosed with congestive heart failure and was prescribed
digoxin. Digoxin is beneficial in this condition because it

*A. reduces the Na gradient across the myocyte membrane.


B. blocks the Ca-ATPase pump.
C. blocks the Ca-induced Ca release channels.
D. increases iCa
E. enhances expulsion of Ca out of the myocyte.

20. Activation of β1 receptors on the myocardial cells of the heart by norepinephrine


leads to all the following EXCEPT

A. increase protein kinase A


B. increase cAMP
C. increased activation of iCa
*D. increased phospholipase C (NE acts on Gs not Gq)
E. increased uptake of calcium into the sarcoplasmic reticulum

21. Which one of the following is most likely to lead to a decrease in contractility of the
heart?

A. exercise
B. dobutamine
C. digoxin
*D. propranolol
E. prazosin

8
22. A 23 year old man has suffered a massive hemorrhage after being savaged by a
mongoose. Fluid is infused in order to increase venous return to the heart. This will cause
the ventricular myocardium to produce more force due in part to:

A. moving the muscle length to the descending limb of the length-tension curve
B. increased end-systolic volume
*C. increased calcium sensitivity to troponin C
D. decreasing afterload
E. increasing afterload

Infusion of fluid after a hemorrhage would increase EDV which would then (by
Starling’s mechanism) increase force generated by the myocardium. In the heart the
actual mechanism is more to do with increasing the sensitivity of the troponin C to
Ca than (as in skeletal muscle) to increase overlap of actin myosin filaments.

23. The opening of the atrio-ventricular valves occurs at about the same time in the
cardiac cycle as the:

*A. beginning of diastole


B. end of isovolumetric contraction
C. first heart sound
D. QRS complex of the ECG
E. beginning of systole

Das könnte Ihnen auch gefallen